Pagina 1 di 2

Densita` densa (own)

Inviato: 01 lug 2009, 11:09
da stefanos
Definizioni.
1. Densita`. Sia $ $~\mathcal{S} \in \mathbb{Z}^+$ $, e sia inoltre $ $~c(n) = \lvert\{m \in S : m \leq n\}\rvert$ $, ovvero la funzione che conta quanti elementi di questo insieme sono minori o uguali a $ $~n$ $. Diciamo che la densita` di $ $~\mathcal{S}$ $ e` $ $~\alpha$ $ se

$ $~\lim_{n\to\infty} \frac{c(n)}{n} = \alpha.$ $

2. Densita` (lol). Diciamo che un insieme $ $~\mathcal{A}\in\mathbb{Q}^+$ $ e` denso in $ $~\mathbb{Q}+$ $ se

$ $~\forall r\in\mathbb{Q}^+, \varepsilon\in\mathbb{R}^+, \exists x\in\mathcal{A} : \lvert r - x \rvert \leq \varepsilon.$ $

Problema.
Siano $ $~\mathcal{S}$ $ un sottoinsieme di $ $~\mathbb{Z}^+$ $ con densita` $ $~1$ $, e $ $~\mathcal{T} = \left\{\frac{m}{n} : m, n\in\mathcal{S}\right\}$ $.

Dimostrare che $ $~\mathcal{T}$ $ e` denso in $ $~\mathbb{Q}^+$ $.

PS: Non sapevo quale sezione fosse piu` appropriata per questo problema...

Edit: anche se in fondo e` la stessa cosa (giusto?) penso che si possa dimostrare che l'ipotesi di questo problema implichi direttamente che T e` denso in R+. Sbaglio?

Inviato: 01 lug 2009, 21:08
da PubTusi
Provo (o almeno ci provo...) una tesi un pò più forte: T contiene tutti i razionali.

Supponiamo esiste un razionale $ \frac mn $ che non appartiene a T.
Allora S non può contenere nessuna coppia di elementi $ (km,kn) $
Allora il complementare di S, che chiamo FB, contiene almeno uno tra km e kn per ogni k. Un elemento può stare massimo in 2 coppie quindi la densita di FB è almeno
$ \frac 1{2m} $ (dove m>n per comodità) ed è diversa da 0.
S ha quindi densita al massimo$ 1-\frac 1{2m} $ perchè è complementare di FB
sisi sono proprio le sue iniziali!
Assurdo

Tu hai fatto così? Ciao!

Inviato: 01 lug 2009, 21:23
da stefanos
Umpfh ma cosi` non vale, il lemma perde il suo significato mistico-trascendentale :? Stupido insieme, perche` doveva contenere un rappresentante di ogni rapporto in Q!? Bah....
Comunque si`, la soluzione funziona, penso :)

La mia era molto piu` articolata :x

Avevo fatto cosi`:

Per assurdo, $ $~\exists r\in\mathbb{Q}^+, \varepsilon\in\mathbb{R}^+ : \forall s, t\in\mathcal{S}, s < (r-\varepsilon) t \vee s > (r+\varepsilon) t$ $, cioe` $ $~\exists a<b\in\mathbb{R}^+ : \forall s, t\in\mathcal{S}, s < a t \vee s > b t$ $. Ma $ $~\forall m\in\mathbb{Z}^+, \exists u\in\mathcal{S} : u>m$ $, altrimenti $ $~\mathcal{S}$ $ sarebbe finito e avrebbe densita` $ $~0$ $, dunque:

$ $~\lim_{n\to\infty} \frac{c(n)}{n} = \lim_{bt\to\infty} \frac{c(bt)}{bt} = \lim_{bt\to\infty}\frac{at}{bt} \leq \lim_{bt\to\infty} \frac{at}{bt} = \frac{a}{b} < 1$ $, assurdo.

Non c'era bisogno di dirlo, l'ho capito appena l'ho letto!

Inviato: 01 lug 2009, 22:49
da Tibor Gallai
Ma perché mi viene da dire "self-owned" anche stavolta? :cry:

Inviato: 02 lug 2009, 12:33
da edriv
A questo punto aggiungiamo le naturali estensioni:

Dimostrare che se un insieme $ S \subset \mathbb{N} $ ha una densità positiva, l'insieme S/S è denso in $ \mathbb Q $.

Un altro modo per dire che un S sottoinsieme di N ha abbastanza elementi è questo:
ci chiediamo se esiste un reale $ \alpha $ tale che, per ogni n sufficientemente grande:
$ \frac{c(n)}{n} \ge \alpha $. Un $ \alpha $ esiste sicuramente... lo 0.
Quindi prendiamo il più grande $ \alpha $ possibile che soddisfi la proprietà, e lo chiamiamo "lower densità" di S.

Dimostrare che, se S ha una "lower densità" positiva, allora non necessariamente S/S è denso in $ \mathbb Q $.

Inviato: 02 lug 2009, 15:15
da kn
edriv ha scritto:Quindi prendiamo il più grande $ \alpha $ possibile che soddisfi la proprietà, e lo chiamiamo "lower densità" di S.
Potrebbe non esistere un $ \alpha $ massimo...
edriv ha scritto:Dimostrare che se un insieme $ S \subset \mathbb{N} $ ha una densità positiva, l'insieme S/S è denso in $ \mathbb Q $.
[...]
Dimostrare che, se S ha una "lower densità" positiva, allora non necessariamente S/S è denso in $ \mathbb Q $.
:shock:

Inviato: 02 lug 2009, 16:27
da edriv
kn ha scritto:
edriv ha scritto:Quindi prendiamo il più grande $ \alpha $ possibile che soddisfi la proprietà, e lo chiamiamo "lower densità" di S.
Potrebbe non esistere un $ \alpha $ massimo...
Già. In tal caso, corriamo ai soliti ripari, e prendiamo il sup. Non cambia un cazzo.

Inviato: 02 lug 2009, 17:03
da kn
edriv ha scritto:Dimostrare che se un insieme $ S \subset \mathbb{N} $ ha una densità positiva, l'insieme S/S è denso in $ \mathbb Q $.
[...]
Dimostrare che, se S ha una "lower densità" positiva, allora non necessariamente S/S è denso in $ \mathbb Q $.
Scusa ma queste due cose non si contraddicono :?:

Inviato: 02 lug 2009, 17:09
da Tibor Gallai
La prima è la densità definita col limite, la seconda è il liminf, sostanzialmente.
Quindi a priori non si contraddicono, ma se credi il contrario, dimostralo. :shock:

Inviato: 02 lug 2009, 17:22
da kn
Ma allora possiamo prendere un sottoinsieme di S con densità $ \displaystyle~\alpha $, che è denso in $ \displaystyle~\mathbb{Q} $. Dov'è che sbaglio? :cry:

Inviato: 02 lug 2009, 17:26
da Tibor Gallai
E' che lo devi dimostrare, gombrendi?? :?

A priori non è così evidente come tu lo possa fare, perché nel 2° caso hai molta più libertà nell'inserire lunghi intervalli di N che non hanno elementi in S...

Inviato: 02 lug 2009, 17:26
da FeddyStra
Ma se la "lower densità" è positiva, allora $ \displaystyle\exists M:\forall n>M:\frac{c(n)}n\ge\alpha>0 $, quindi $ \displaystyle\lim_{n\to\infty}\frac{c(n)}n\ge\alpha>0 $.
Ovvero "lower densità" positiva implica densità positiva, che a sua volta implica $ S/S $ denso su $ \mathbb Q^+ $.

Inviato: 02 lug 2009, 17:28
da Tibor Gallai
Giaquinta, Modica ha scritto:3. I limiti non sempre esistono.

Inviato: 02 lug 2009, 17:32
da FeddyStra
Right! :wink: Ecco l'inghippo!

Inviato: 02 lug 2009, 17:37
da kn
Allora costruisco un sottoinsieme S' di S in questo modo:
$ \displaystyle~S'=S/\{x\} $, con x il più piccolo intero $ \displaystyle~>M:\frac{c(x)}x>\alpha $ (l'M è quello di FeddyStra)
Iterando questo processo all'infinito (con $ \displaystyle~c(\cdot) $ riferita ai nuovi sottoinsiemi) ottengo un sottoinsieme di S con densità $ ~\alpha $...
Abbi pazienza ma continuo a non capire... :cry: